Tomando el límite continuo de las teorías de calibre U(N)U(N)U(N)

Me gustaría llamar su atención sobre el apéndice C en la página 38 de este trabajo .

  • La ecuacion C .2 parece estar evaluando la suma R x R ( tu metro ) en la ecuación 3.16 de este trabajo . Entiendo que tu tu ( norte C ) grupo y luego con norte F (¿fundamental?) hipermultipletos parece que aparentemente se afirma que lo siguiente es cierto,

R x R ( tu metro ) = 2 norte F [ T r tu metro + T r tu metro ]

¿Alguien puede ayudar a derivar lo anterior?

  • La primera línea de la ecuación C .4 parece ser lo mismo que la ecuación 5.3 de este documento vinculado anterior , pero no del todo...

  • Estaba bastante confundido con la idea en la ecuación C .5 y C .6 de pensar los valores propios de los tu ( norte C ) grupo en el círculo dado por una función de distribución ρ ( θ ) 0 con una misteriosa normalización como en la ecuación C .5 .

    Me alegraría si alguien puede ayudar a entender cómo se determinó la ecuación C.6. ¡Parece que aquí se omitieron muchos pasos que me resultaron difíciles de restablecer!

  • A juzgar por este documento , parece que la ecuación original más general 2.43 en la página 17 tiene dos límites posibles como en la ecuación 2.45 en la página 17 cuando hay C campos de materia en el adjunto y norte C o ecuación 2.48 en el límite Veneziano con campos de materia fundamental.

    Me gustaría saber la derivación/referencia de las ecuaciones 2.45 y 2.48 citadas anteriormente.

    Supongo que este es uno de los medios más educativos para tomar el límite Veneziano en el que se supone que funciona AdS/CFT.

Respuestas (1)

El personaje x R : GRAMO C de una representación R es definido por x R ( tu ) = T r R ( tu ) , es decir, tomando el rastro de tu en la representación R . Véase, por ejemplo, el Apéndice A de Aharony et al. . Entonces la ecuación que escribe parece razonable: presumiblemente hay norte F hipermultipletes, cada uno con campos fundamentales y anti-fundamentales que dan T r ( tu metro ) y T r ( tu metro ) respectivamente en la suma de R . Se toma la traza de la matriz. tu porque los campos están en lo fundamental.

El factor de 2 debería provenir de los detalles de qué representaciones exactamente se están sumando, el contenido de campo de los hipermultipletes, etc., que no traté de seguir.

Ahora, con respecto a la distribución de valores propios que aparece en (C.5), déjame darte una explicación física de dónde viene esto. Consideran la termodinámica de una teoría sobre una esfera, por lo que la topología es la de una esfera por S 1 -- el tiempo compacto euclidiano. En este caso, hay un modo cero del campo del indicador, que proviene del hecho de que puede activar un campo constante en la dirección térmica. Este modo no se puede eliminar mediante una transformación de calibre (en general), por lo que en una integral de ruta debe integrarse sobre ella. Esto se explica, por ejemplo, en la sección 4.1 de Aharony et al . Puede usar una transformación de calibre para diagonalizar esta matriz de modo cero, por lo que le quedan los valores propios discretos de la matriz para integrar. También puede mostrar que estos valores propios viven en un círculo, porque puede cambiarlos por 2 π (en alguna normalización) usando una transformación de indicador, por lo que debe integrarlos en un círculo. en el grande norte límite, tiene un número infinito de tales valores propios, pero aún están restringidos a vivir en un círculo. Entonces necesitas describirlos usando una densidad, y eso es lo que ellos llaman la distribución ρ .

Editar: agregar explicación de (C.6)

Para derivar (C.6), considere primero la suma

i porque ( norte α i ) = norte 0 2 π ρ ( θ ) porque ( norte θ )

dónde ρ ( θ ) = 1 norte i d ( θ α i ) . Darse cuenta de ρ tiene la normalización correcta.

al tomar norte las funciones delta en ρ se volverá muy denso, y podremos aproximarnos ρ por una función suave, cuyo valor en θ depende de la densidad de las funciones delta a lo largo del pequeño intervalo [ θ , θ + ϵ ] . Esta será una buena aproximación porque la función porque ( norte θ ) sobre el que estamos integrando no variará mucho en estos intervalos, por lo que promediarlo en lugar de muestrearlo no cambiará mucho el resultado. Cuando llegamos al límite norte = esto ya no será una aproximación, porque las funciones delta se volverán continuas.

Así que esto explica la derivación del segundo término en (C.6). En cuanto al primer término, la idea es similar excepto que tienes dos integrales sobre θ , θ correspondiente a la i , j sumas Las contribuciones de la i = j términos ( θ = θ en la integral) son sublíderes en la gran norte límite: escalan como norte mientras que el resto escala como norte 2 , por lo que se descuidan.

Ahora, reemplaza porque ( norte ( θ θ ) ) = porque ( norte θ ) porque ( norte θ ) + pecado ( norte θ ) pecado ( norte θ ) . Observe que en la integral de trayectoria sobre ρ puede considerar solo distribuciones de valores propios que son simétricas bajo θ θ , porque el integrando original es invariante bajo esto. Esto se menciona explícitamente, por ejemplo, en Schnitzer , quien hace el mismo cálculo. Entonces esto significa que d θ ρ pecado ( norte θ ) = 0 . Creo que debe quedar claro a partir de este punto.

@ Guy Gur-Ari Cada uno de los norte F hipermultipletos creo que tiene 2 norte = 2 multipletes quirales uno en el adjunto y el otro en el adjunto conjugado (= adjunto) del grupo calibre. Ahora tenemos x a d j = x a d j ¯ = x F tu norte d × x a norte t i F tu norte d . Así que cada hipermultiplete debería haber contribuido con un 2 x F tu norte d × x a norte t i F tu norte d ..¿bien?
Puedo ver este argumento general de la necesidad de describir el valor propio de un tu ( norte ) matriz por una densidad en el círculo, pero es la reescritura específica de eso como en la ecuación C.6 lo que me está desconcertando. ¡Sería de gran ayuda si pudieras escribir en unas pocas líneas cómo se obtuvo esa ecuación!
Los multipletes importantes que consideran en el apéndice C están en lo fundamental, como escriben al principio del apartado 3.2 (un poco antes de referirse al apéndice). Agregué la explicación de (C.6) a la respuesta.
¡Gracias por las actualizaciones! Creo que me estoy perdiendo algo aquí. Al comienzo de la sección 2.2 (página 7), ¿no dicen que los 2 multipletes quirales N=2 dentro del hipermultiplete N=3 están en representaciones conjugadas del grupo de calibre? Entonces, ¿es incorrecto pensar que estos 2 N=2 componentes quirales están en A d j = A d j ¯ del grupo de calibre? Si el norte F hipermultipletes están en lo fundamental entonces cuál es la representación en la que los 2 norte = 2 ¿En qué caen los multipletes quirales? ¡Será genial si puedes aclarar este punto!
Como dicen, en el norte = 3 En teoría, puede tener hipermultipletes, y cada hipermultiplete tiene dos multipletes quirales: uno en un representante R y uno en R ¯ . No es necesario que R es el adjunto. Un poco más abajo del lugar que mencionas, consideran varias posibilidades para R , sólo uno de los cuales es el adjunto. Si dice que un hipermultiplete es 'fundamental' (que creo que no es una declaración precisa aunque su significado es claro), significa que tiene un multiplete quiral en el fundamental y otro en el anti-fundamental.
¡Gracias por la explicación! ¿Me puede dar una referencia donde estos multipletes sin masa de norte = 2 y norte = 3 supersimetría en 2 + 1 las dimensiones se derivan de cero? Me gusta si puede ayudar con esta pregunta mía anterior, physics.stackexchange.com/questions/33023/… ¡Eso sería de gran ayuda!
No tengo referencia pero la historia es similar a los multipletes en d = 4 , que se derivan de cualquier libro de texto sobre SUSY. Puedes repetir este ejercicio en d = 3 .
No estoy tan seguro. En d=4, lo especial que sucede es que la anticonmutación entre las Q es diagonal y, por lo tanto, la mitad va a 0 y la otra mitad funciona como el álgebra de Clifford. En d=3 esta simplificación no existe por las peculiaridades de las matrices Gamma en esa dimensión. Por lo tanto, me gustaría saber si existe una referencia estándar que explique la construcción del multiplete en d=3.
Dije que es similar, no es lo mismo. Me parece que esta discusión se ha desviado de la pregunta original. ¿Por qué no publicarlo como una nueva pregunta?
Ya había hecho la pregunta por separado como se vincula desde mi comentario anterior. Ha quedado sin respuesta. Por cierto, te he otorgado la recompensa. ¡Gracias por la ayuda! ¡Espero verte en esa otra pregunta mía también!
He publicado una versión un poco más editada de esa pregunta, physics.stackexchange.com/questions/33023/… Sería genial contar con su ayuda.
Creo que mi pregunta anterior no se redactó correctamente y, con suerte, hice una mejor declaración aquí, physics.stackexchange.com/questions/34192/… ¡Me alegraría verte allí!
@GuyGur-Ari Algunos pensamientos recientes me trajeron de vuelta a esta discusión contigo. Por tu construcción ρ norte es real, y por lo tanto, cuando pasan de 5.6 a 5.10 en el documento arxiv.org/pdf/hep-th/0310285v6.pdf , deberían haber obtenido una raíz cuadrada, pero no lo hacen, parece que su \rho_n es complejo: ¿puede explicar amablemente este punto?